User avatar
 
noah
Thanks Received: 1192
Atticus Finch
Atticus Finch
 
Posts: 1541
Joined: February 11th, 2009
 
This post thanked 3 times.
 
 

Q13 - Some doctors believe that a certain

by noah Fri Dec 31, 1999 8:00 pm

The argument here is that a drug purported to shorten bouts of vertigo does not actually do so. The evidence? That during a 3-month shortage of the drug, the duration of patient's bouts of vertigo did not change.

There are two assumptions here that you may have noticed (and perhaps others):
1. That the drug's affect appear or disappear within 3-months. Maybe it takes 5 months for the drug to build up enough presence in one's system to affect a change, and a similar length of time to dissipate.
2. That the average duration of the bouts of vertigo is an accurate measurement -- perhaps there were other factors in the environment that lengthened duration for others?

(A) plays on the issue in #1 above. The argument can be stripped down to:
No change --> Not effective, and (A) provides the contrapositive: Effective --> Change

If we were to negate (A) and state that if the drug were effective we would not see a change, then the argument becomes invalid. The overall efficacy and the change witnessed after a shortage would be "unhooked."

As for the incorrect answers:

(B) is tempting. If provides Change --> Effective, which is the reverse of the contrapositive, and so it does not help us. Put simply, we are interested in the effects of there not being a change, not what happens if there were a change.

(C) may be true -- perhaps 4 months would be a better test. But because a fact is less useful than another does not mean it is not true or useful. A DNA match might be better than a fingerprint, but that doesn't mean that a fingerprint is useless or not enough to convict.

(D) is irrelevant. Even if those other factors are relevant, we don't know that the drug is irrelevant. When we negate (D)--diet or smoking does change the duration of vertigo--it actually bolsters the conclusion that the drug has no effect.

(E) is similarly irrelevant. Whether there are significant factors other than the drug does not affect whether the drug is effective.


#officialexplanation
 
lhermary
Thanks Received: 10
Atticus Finch
Atticus Finch
 
Posts: 160
Joined: April 09th, 2011
 
 
 

Re: Q13 - Some doctors believe that a certain

by lhermary Tue May 08, 2012 6:09 pm

Can you go into more detail as to why C is wrong?

Conclusion: The drug has no effect on the duration of vertigo
Why: because in a 3 month study there was no significant change in the average duration of vertigo

But what about if the duration was 4 months? Wouldn't that change the accuracy of the conclusion?

Thanks
User avatar
 
ManhattanPrepLSAT1
Thanks Received: 1909
Atticus Finch
Atticus Finch
 
Posts: 2851
Joined: October 07th, 2009
 
This post thanked 1 time.
 
 

Re: Q13 - Some doctors believe that a certain

by ManhattanPrepLSAT1 Sun May 13, 2012 4:35 pm

lhermary Wrote:But what about if the duration was 4 months? Wouldn't that change the accuracy of the conclusion?

Exactly. That's why we need to assume that the 3 month period would allow for the assessment of whether the drug was having an effect on vertigo as expressed in answer choice (A).

The issue with answer choice (C) is the relative comparison between a study of just 3 months and a study that would take longer than 3 months. As Noah pointed out, the possibility that a 3 month study may not be as good as a longer study would not discredit the argument. The negation of answer choice (C) does not suggest that a 3 month study would not work (which is what the negation of answer choice [A] says), but rather that a longer study would be even better. Maybe they're both effective in determining the drug's impact, but that a longer study would provide even more reliable/useful information.

Does that answer your question?
 
shirando21
Thanks Received: 16
Atticus Finch
Atticus Finch
 
Posts: 280
Joined: July 18th, 2012
 
 
 

Re: Q13 - Some doctors believe that a certain

by shirando21 Wed Aug 29, 2012 12:46 pm

noah Wrote:The argument here is that a drug purported to shorten bouts of vertigo does not actually do so. The evidence? That during a 3-month shortage of the drug, the duration of patient's bouts of vertigo did not change.

There are two assumptions here that you may have noticed (and perhaps others):
1. That the drug's affect appear or disappear within 3-months. Maybe it takes 5 months for the drug to build up enough presence in one's system to affect a change, and a similar length of time to dissipate.
2. That the average duration of the bouts of vertigo is an accurate measurement -- perhaps it was a small sample and there were other factors in the environment that lengthened duration for others?

(A) plays on the issue in #1 above. The argument can be stripped down to:
No change --> Not effective, and (A) provides the contrapositive: Effective --> Change

If we were to negate (A) and state that if the drug were effective we would not see a change, then the argument becomes invalid. The overall efficacy and the change witnessed after a shortage would be "unhooked."

As for the incorrect answers:

(B) is tempting. If provides Change --> Effective, which is the reverse of the contrapositive, and so it does not help us. Put simply, we are interested in the effects of there not being a change, not what happens if there were a change.

(C) may be true -- perhaps 4 months would be a better test. But because a fact is less useful than another does not mean it is not true or useful. A DNA match might be better than a fingerprint, but that doesn't mean that a fingerprint is useless or not enough to convict.

(D) is irrelevant. Even if those other factors are relevant, we don't know that the drug is irrelevant.

(E) is similarly irrelevant. Whether there are significant factors other than the drug does not affect whether the drug is effective.


Is A also a sufficient assumption?
User avatar
 
noah
Thanks Received: 1192
Atticus Finch
Atticus Finch
 
Posts: 1541
Joined: February 11th, 2009
 
 
 

Re: Q13 - Some doctors believe that a certain

by noah Wed Aug 29, 2012 12:54 pm

shirando21 Wrote:Is A also a sufficient assumption?


Yes, it seems to make the argument airtight. Tell me if you see some reason it doesn't.
 
shirando21
Thanks Received: 16
Atticus Finch
Atticus Finch
 
Posts: 280
Joined: July 18th, 2012
 
 
 

Re: Q13 - Some doctors believe that a certain

by shirando21 Wed Aug 29, 2012 3:02 pm

noah Wrote:
shirando21 Wrote:Is A also a sufficient assumption?


Yes, it seems to make the argument airtight. Tell me if you see some reason it doesn't.


So is the rule like this:

1. If the question asks for a necessary assumption, an answer that is a necessary assumption or a sufficient assumption will do the job.

2. If the question asks for a sufficient assumption, only an answer that is a sufficient assumption will do, an answer that is only a necessary assumption will be wrong answer.
User avatar
 
noah
Thanks Received: 1192
Atticus Finch
Atticus Finch
 
Posts: 1541
Joined: February 11th, 2009
 
This post thanked 1 time.
 
 

Re: Q13 - Some doctors believe that a certain

by noah Wed Aug 29, 2012 3:41 pm

shirando21 Wrote:So is the rule like this:

1. If the question asks for a necessary assumption, an answer that is a necessary assumption or a sufficient assumption will do the job.

2. If the question asks for a sufficient assumption, only an answer that is a sufficient assumption will do, an answer that is only a necessary assumption will be wrong answer.

Almost.

1. If the question asks for a necessary assumption, only an answer that is a necessary assumption (including those that are both a necessary and sufficient assumption) will do the job.

2. If the question asks for a sufficient assumption, only an answer that is a sufficient assumption will do, an answer that is only a necessary assumption will be wrong answer.
 
kpopstar123
Thanks Received: 1
Forum Guests
 
Posts: 14
Joined: October 24th, 2012
 
 
 

Re: Q13 - Some doctors believe that a certain

by kpopstar123 Tue Nov 20, 2012 12:04 am

can someone tell me what does it mean from however... to the conclusion?

I mean I don't understand what role does "shortage play" in this argument.
User avatar
 
noah
Thanks Received: 1192
Atticus Finch
Atticus Finch
 
Posts: 1541
Joined: February 11th, 2009
 
 
 

Re: Q13 - Some doctors believe that a certain

by noah Tue Nov 20, 2012 5:21 pm

kpopstar123 Wrote:can someone tell me what does it mean from however... to the conclusion?

I mean I don't understand what role does "shortage play" in this argument.

It means that when there was no drug for 3 months, there was no change in how long vertigo lasted. Thus, the conclusion that the drug doesn't affect how long vertigo lasts.

(A) fits in because what if it takes 5 months for the drug to wear off? In order to conclude that the drug has no effect we need the premise to be indicative of the drug's effect--so we need 3 months to be a relevant time frame.

Imagine the argument was that the patients took the drug 5 hours late and there was no change in how long vertigo lasted. We'd think "5 hours, that might not be long enough!" Well, who is to say that 3 months is long enough?
 
claudia.minoiu
Thanks Received: 0
Vinny Gambini
Vinny Gambini
 
Posts: 6
Joined: April 09th, 2013
 
 
 

Re: Q13 - Some doctors believe that a certain

by claudia.minoiu Tue Apr 23, 2013 1:41 pm

I was going to choose A as well, but was deterred by the use of language "significant change" while the language in the argument is less drastic: "some doctors... claiming that the average duration of [.] has DECREASED since the drug was introduced". Doesn't say anything about a SIGNIFICANT decrease.

Anyway, I thought this was a tough one :(
 
wgutx08
Thanks Received: 8
Elle Woods
Elle Woods
 
Posts: 52
Joined: June 09th, 2013
 
 
 

Re: Q13 - Some doctors believe that a certain

by wgutx08 Tue Aug 13, 2013 6:45 pm

noah Wrote:
shirando21 Wrote:So is the rule like this:

1. If the question asks for a necessary assumption, an answer that is a necessary assumption or a sufficient assumption will do the job.

2. If the question asks for a sufficient assumption, only an answer that is a sufficient assumption will do, an answer that is only a necessary assumption will be wrong answer.

Almost.

1. If the question asks for a necessary assumption, only an answer that is a necessary assumption (including those that are both a necessary and sufficient assumption) will do the job.

2. If the question asks for a sufficient assumption, only an answer that is a sufficient assumption will do, an answer that is only a necessary assumption will be wrong answer.


I had very similar questions...

For 1: But there are so few situations where the assumption is both sufficient and necessary. It is here in this question, because A basically just restates the conclusion in contrapositive, so naturally it is both nec and suff.

However, the majorities of the nec assumption answers are only sufficient not necessary (like Q9 in this section. Surely proportional is not required for the conclusion--- say, the total population decreased even more, like 5/6! A real necessary assumption would be sth. like "the decrease in total population is not typically much smaller than the decrease in female population".)

and then you have Qs like 52.3.7, where I feel A is neither necessary nor sufficient. If I'm wrong here, I would really appreciate to know why.

:? :? :( :(
User avatar
 
noah
Thanks Received: 1192
Atticus Finch
Atticus Finch
 
Posts: 1541
Joined: February 11th, 2009
 
 
 

Re: Q13 - Some doctors believe that a certain

by noah Tue Aug 13, 2013 6:51 pm

wgutx08 Wrote:I had very similar questions...

For 1: But there are so few situations where the assumption is both sufficient and necessary. It is here in this question, because A basically just restates the conclusion in contrapositive, so naturally it is both nec and suff.

However, the majorities of the nec assumption answers are only sufficient not necessary (like Q9 in this section. Surely proportional is not required for the conclusion--- say, the total population decreased even more, like 5/6! A real necessary assumption would be sth. like "the decrease in total population is not typically much smaller than the decrease in female population".)

and then you have Qs like 52.3.7, where I feel A is neither necessary nor sufficient. If I'm wrong here, I would really appreciate to know why.

:? :? :( :(

The answers to LSAT necessary questions are definitely necessary. Once in a while they're also sufficient. (I think you might have mis-typed above?)

Answers to what I call "basic assumption" question--usually phrased as "the argument assumes that..." are both necc. and suff. (and you can just treat those questions as necc. assumption ones, but they're generally quite easy, so don't sweat it).

For the other questions you ask about, go ahead and read the explanations in those threads and see if your idea is discussed. If not, go ahead and post your questions. (Sorry to do that, but otherwise our forums becomes a zoo with people asking things anywhere, dogs marrying cats, etc.)
 
wgutx08
Thanks Received: 8
Elle Woods
Elle Woods
 
Posts: 52
Joined: June 09th, 2013
 
 
 

Re: Q13 - Some doctors believe that a certain

by wgutx08 Tue Aug 13, 2013 7:06 pm

Many thanks Noah for the quick reply!

Sorry about the ectopic posting, but I meant to have these 2 other Qs as examples for my somewhat general question. Totally see what you mean though (NOOO, don't want to see dogs marrying cats at all), I already posted the Q for 52.3.7 in the right place...

thanks
 
lsatodyssey
Thanks Received: 0
Forum Guests
 
Posts: 8
Joined: November 05th, 2012
 
 
 

Re: Q13 - Some doctors believe that a certain

by lsatodyssey Sat Aug 17, 2013 8:03 pm

"(D) is irrelevant. Even if those other factors are relevant, we don't know that the drug is irrelevant."

If we negate D, doesn't that impair our ability to analyze whether the drug had any effect? Therefore the argument would need to defend against this possibility?

I was a little confused by this. Thanks!
User avatar
 
noah
Thanks Received: 1192
Atticus Finch
Atticus Finch
 
Posts: 1541
Joined: February 11th, 2009
 
This post thanked 1 time.
 
 

Re: Q13 - Some doctors believe that a certain

by noah Mon Aug 19, 2013 11:03 am

akam41 Wrote:"(D) is irrelevant. Even if those other factors are relevant, we don't know that the drug is irrelevant."

If we negate D, doesn't that impair our ability to analyze whether the drug had any effect? Therefore the argument would need to defend against this possibility?

I was a little confused by this. Thanks!

Good question!

If we negate (D) and it turns out that going vegan explains some of the reduction in vertigo bout duration could we still conclude that the drug has no effect? Yes! In fact, this helps. This is why we don't care if the other factors are relevant--we're only wondering whether we can prove that the drug is IRrelevant.

It's so easy to get turned around--you got it?
 
amil91
Thanks Received: 5
Elle Woods
Elle Woods
 
Posts: 59
Joined: August 02nd, 2013
 
 
 

Re: Q13 - Some doctors believe that a certain

by amil91 Fri Oct 25, 2013 11:13 pm

noah Wrote:
akam41 Wrote:"(D) is irrelevant. Even if those other factors are relevant, we don't know that the drug is irrelevant."

If we negate D, doesn't that impair our ability to analyze whether the drug had any effect? Therefore the argument would need to defend against this possibility?

I was a little confused by this. Thanks!

Good question!

If we negate (D) and it turns out that going vegan explains some of the reduction in vertigo bout duration could we still conclude that the drug has no effect? Yes! In fact, this helps. This is why we don't care if the other factors are relevant--we're only wondering whether we can prove that the drug is IRrelevant.

It's so easy to get turned around--you got it?

But if something else is relevant doesn't that shed doubt on the validity of the conclusion? To me D was an example of an alternative explanation for the duration of vertigo decreasing that was negated, and it was my understanding that necessary assumptions can simply negate an alternative explanation. When I was doing this problem, I was down to A and D, so I tried negating each one. I negated D to changes in diet and smoking are responsible for any change.... and I negated A to if a drug made a difference in the duration of vertigo, a three-month shortage of that drug would not have cause.... To me if D is negated, and diet and smoking habit changes are responsible for any change, then who's to say the average duration of vertigo didn't change during this 3 month period because people changed their diet or smoking habits in a manner that off-set the change that was occurring due to the drug before the shortage occurred? To me my negation of A was simply confusing and did not seem to dismantle the argument. Perhaps I negated it incorrectly? I ultimately picked D because I thought A was a sufficient, but not necessary assumption. Any help would be appreciated.
User avatar
 
tommywallach
Thanks Received: 468
Atticus Finch
Atticus Finch
 
Posts: 1041
Joined: August 11th, 2009
 
 
 

Re: Q13 - Some doctors believe that a certain

by tommywallach Mon Oct 28, 2013 1:24 am

Hey Amil,

You're missing the "since the introduction of the drug." In that case, the drug has already had an effect, and now any OTHER effects will be from diet and smoking. However, this does not wreck the conclusion that the drug doesn't affect vertigo (and the correctly negated assumption should DESTROY the argument!

-t
Tommy Wallach
Manhattan LSAT Instructor
twallach@manhattanprep.com
Image
 
LSAT taker
Thanks Received: 0
Vinny Gambini
Vinny Gambini
 
Posts: 1
Joined: October 03rd, 2015
 
 
 

Re: Q13 - Some doctors believe that a certain

by LSAT taker Sat Oct 24, 2015 8:37 pm

tommywallach Wrote:Hey Amil,

You're missing the "since the introduction of the drug." In that case, the drug has already had an effect, and now any OTHER effects will be from diet and smoking. However, this does not wreck the conclusion that the drug doesn't affect vertigo (and the correctly negated assumption should DESTROY the argument!

-t


I am wondering whether it matters that the prompt also refers to the time-frame "since the introduction of the drug" in relation to the claimed effect of the drug. Specifically, "...claiming that the average duration of vertigo for people who suffer from it has decreased since the drug was introduced". So, the effect of the drug does not appear to be preceding the effects of diet / smoking habits. If we negate (D) taking into account the "since the introduction of the drug", it is not clear if the average duration of vertigo didn't change during the 3 month period because people changed their diet / smoking habits in a manner that offset any effect of the drug, as the poster above pointed out. Accordingly, I am not able to rule this out as a possible necessary assumption for the argument.

Any insight would be much appreciated, thank you!
 
at9037
Thanks Received: 0
Vinny Gambini
Vinny Gambini
 
Posts: 14
Joined: September 08th, 2015
 
 
 

Re: Q13 - Some doctors believe that a certain

by at9037 Fri Jun 30, 2017 11:52 pm

I still can't seem to understand why B cannot be the answer. Wouldn't negating B also invalidate the conclusion? Also, since the argument focuses on the average duration of vertigo wouldn't it be necessary for no change to occur in the average duration of vertigo?

I can understand why A is the answer, but I cannot see why B isn't.

Also, could you please explain how A also qualifies as a sufficient assumption?
 
andrewgong01
Thanks Received: 61
Atticus Finch
Atticus Finch
 
Posts: 289
Joined: October 31st, 2016
 
 
 

Re: Q13 - Some doctors believe that a certain

by andrewgong01 Fri Jul 14, 2017 7:07 pm

at9037 Wrote:I still can't seem to understand why B cannot be the answer. Wouldn't negating B also invalidate the conclusion? Also, since the argument focuses on the average duration of vertigo wouldn't it be necessary for no change to occur in the average duration of vertigo?

I can understand why A is the answer, but I cannot see why B isn't.

Also, could you please explain how A also qualifies as a sufficient assumption?


B in conditional language is "If any changes in duration since the drug, then drug had an effect". This is very strong in terms of language. If there were any changes since the drug started triggers the conditional. Were there any changes? Yes, sentence 1 does say that there were changes in duration. What does this mean according to the answer choice? It means that the drug had an effect but that goes against the conclusion, which is the drug had no effect. This would be a weakener

A is a SA too because in conditional language it is, put succinctly, "If drug made difference in duration of vertigo, then 3 month caused a change in vertigo. Its contrapositive is 'If 3 month no change in vertigo, then no difference in duration due to drug".

Based of the stimulus we have triggered the 3 month no change in vertigo ( second to last sentence) and hence since we triggered it we get to the necessary condition - no difference in duration due to drug. This allows us to conclude that the drug was not efficacious , which is what the stimulus did to. Hence it is a SA as it makes the argument air tight thanks to the conditional language.